Inference CR

This topic has expert replies
Master | Next Rank: 500 Posts
Posts: 280
Joined: Tue Sep 30, 2008 4:18 am
Thanked: 5 times
GMAT Score:610

Inference CR

by Jatinder » Sun Nov 16, 2008 8:16 am
When the rate of inflation exceeds the rate of return on the most profitable investment available, the difference between those two rates will be the percentage by which, at a minimum, the value of any investment will decline.If in such a circumstance the value of any particular investment declines by more than that percentage, it must be true that :................

WHich one of the following logically completes the argument below?
a. the rate of inflation has risen
b.the investment in question is becoming less profitable
c.the investment in question is less profitable than the most profitable investment available
d.the rate of return on the most profitable investment available has declined
e.there has been a change in which particular investment happens to be most profitable available
Keep flying

Legendary Member
Posts: 683
Joined: Tue Jul 22, 2008 1:58 pm
Location: Dubai
Thanked: 73 times
Followed by:2 members

by mals24 » Sun Nov 16, 2008 9:15 am
OMG What is this question!!!!

Alright Ill give it a shot but I don't know if my reasoning is right or no.

The argument says that the difference between Rate of Inflation (RoI) and Rate of return (RoR) on the most profitable investment is the percentage by which, at minimum, the value of an investment can decline.

Say for instance there are two companies.
Company A is the most profitable company whose RoR = 10%
Company B where we make our investment.
Suppose we have $100,000 to invest.

RoI = 20%
RoR = 10%
Difference = 10%
Hence this 10% is the least a value can decline.

What if value of our investment in company B declines by 11%?
In company A, our investment’s value declines by 10% = 10,000
The value of our investment in A would have been 90,000

In company B, our investment’s value declines by 11% = 11,000
The value of our investment is 89,000.

Hence our investment (in company B) is less profitable than investment in company A.
Only option C supports this.
So my vote for C.

Master | Next Rank: 500 Posts
Posts: 280
Joined: Tue Sep 30, 2008 4:18 am
Thanked: 5 times
GMAT Score:610

by Jatinder » Sun Nov 16, 2008 10:45 pm
mals24 wrote:OMG What is this question!!!!

Alright Ill give it a shot but I don't know if my reasoning is right or no.

The argument says that the difference between Rate of Inflation (RoI) and Rate of return (RoR) on the most profitable investment is the percentage by which, at minimum, the value of an investment can decline.

Say for instance there are two companies.
Company A is the most profitable company whose RoR = 10%
Company B where we make our investment.
Suppose we have $100,000 to invest.

RoI = 20%
RoR = 10%
Difference = 10%
Hence this 10% is the least a value can decline.

What if value of our investment in company B declines by 11%?
In company A, our investment’s value declines by 10% = 10,000
The value of our investment in A would have been 90,000

In company B, our investment’s value declines by 11% = 11,000
The value of our investment is 89,000.

Hence our investment (in company B) is less profitable than investment in company A.
Only option C supports this.
So my vote for C.
Don't you think that B is just the outcome of C.
that is if C is true, B has to be true
Keep flying

User avatar
Senior | Next Rank: 100 Posts
Posts: 50
Joined: Fri Oct 24, 2008 12:25 am
Location: New York, US

by slash » Mon Nov 17, 2008 1:17 am
My option goes with A

Im not very sure that I'm right...but wanted to try

The diffrence between rate of Inflation and rate of return is the percentage by which value of investment will decline.

If the investment declines further it must be tru that rate of inflation has risen.

Legendary Member
Posts: 683
Joined: Tue Jul 22, 2008 1:58 pm
Location: Dubai
Thanked: 73 times
Followed by:2 members

by mals24 » Mon Nov 17, 2008 3:34 am
@Jatinder

I thought the same too. I'm actually still confused between B & C, I went for C because I thought it was better supported.
Also we don't know the investment is becoming less profitable in general. Its profitability is definitely lower than the most profitable investment. Probably its the 2nd most profitable investment in the market. Probably the other investments profitability has declined even further. In this case our investment is still profitable compared to the other investments in the market right.

This is the logic I used to choose between C & B.

Legendary Member
Posts: 683
Joined: Tue Jul 22, 2008 1:58 pm
Location: Dubai
Thanked: 73 times
Followed by:2 members

by mals24 » Mon Nov 17, 2008 3:34 am
@slash
The % is the minimum a value of an investment can decline. Now the argument states that the investment value declined by more than that %. Which means the % remains the same, only our investment’s value has changed. Now we don’t know what caused this change. We only know that it declined. So we cannot assume that the RoR declined or RoI increased or whether the investment from which we calculate the RoR has changed. Hence A, D & E are ruled out since they are not supported by the argument.

Master | Next Rank: 500 Posts
Posts: 280
Joined: Tue Sep 30, 2008 4:18 am
Thanked: 5 times
GMAT Score:610

by Jatinder » Mon Nov 17, 2008 6:52 am
thanks guys, although question still looks lil ambiqous
OA is C :-)
source: 800score.com
Keep flying

User avatar
Legendary Member
Posts: 871
Joined: Wed Aug 13, 2008 7:48 am
Thanked: 48 times

by stop@800 » Tue Nov 18, 2008 1:51 am
B assumes that the investment itself is declining which is not inline with what has been said in premise.

C saya that choice is incorrect so correctly completes the argument.

Master | Next Rank: 500 Posts
Posts: 100
Joined: Tue Aug 05, 2014 3:58 am

by vanessa.m » Fri May 13, 2016 1:55 am
C seems to be the most logical one

Master | Next Rank: 500 Posts
Posts: 197
Joined: Tue Apr 22, 2014 4:03 am

by nicolette » Fri May 13, 2016 1:56 am
Cannot decide between C and B. Can anyone brake down these two choices for me please